- Fri Jan 21, 2011 12:00 am
#23995
Complete Question Explanation
Weaken. The correct answer choice is (E)
The prediction presented here is a simple one: that if oil prices fall by half, then gasoline prices will fall by half. The question stem which follows requires us to find the answer choice which calls this prediction into question. The correct answer will give reason to doubt the associated fall in gas prices.
Answer choice (A): A reduced use of gasoline by “some” consumers would not help to refute the prediction that falling oil prices would be accompanied by proportionate falling gas prices, so this answer choice is incorrect.
Answer choice (B): This choice would actually strengthen the argument in the stimulus, ruling out one factor that might throw off the proportionality between oil prices and gas prices. That is, if we can count on the gas makers to avoid increasing their profit margins, then it seems more likely that a drop in oil prices would be accompanies by a proportionate drop in gas prices.
Answer choice (C): The presence of a competitive gas market would not refute the prediction made in the stimulus. In fact, this choice strengthens the argument: if many gas companies are competing, then this increases the likelihood that a drop in oil prices would translate into a similar drop in gas prices (rather than an increase in gas makers’ profits, for example).
Answer choice (D): The information presented in this choice is extremely vague, and fails to refute the prediction presented in the stimulus. Basically, this information give us no reason to question the argument that a 50% drop in oil prices will be accompanied by a 50% drop in gas prices.
Answer choice (E): This is the correct answer choice. Here we are provided with a list of fixed costs which come into play regardless of oil prices. The presence of such costs means that gas prices consider much more than just the price of oil. This makes it unlikely that the drop in oil prices will be mirrored by a drop in gas prices.
Weaken. The correct answer choice is (E)
The prediction presented here is a simple one: that if oil prices fall by half, then gasoline prices will fall by half. The question stem which follows requires us to find the answer choice which calls this prediction into question. The correct answer will give reason to doubt the associated fall in gas prices.
Answer choice (A): A reduced use of gasoline by “some” consumers would not help to refute the prediction that falling oil prices would be accompanied by proportionate falling gas prices, so this answer choice is incorrect.
Answer choice (B): This choice would actually strengthen the argument in the stimulus, ruling out one factor that might throw off the proportionality between oil prices and gas prices. That is, if we can count on the gas makers to avoid increasing their profit margins, then it seems more likely that a drop in oil prices would be accompanies by a proportionate drop in gas prices.
Answer choice (C): The presence of a competitive gas market would not refute the prediction made in the stimulus. In fact, this choice strengthens the argument: if many gas companies are competing, then this increases the likelihood that a drop in oil prices would translate into a similar drop in gas prices (rather than an increase in gas makers’ profits, for example).
Answer choice (D): The information presented in this choice is extremely vague, and fails to refute the prediction presented in the stimulus. Basically, this information give us no reason to question the argument that a 50% drop in oil prices will be accompanied by a 50% drop in gas prices.
Answer choice (E): This is the correct answer choice. Here we are provided with a list of fixed costs which come into play regardless of oil prices. The presence of such costs means that gas prices consider much more than just the price of oil. This makes it unlikely that the drop in oil prices will be mirrored by a drop in gas prices.